Integral in spherical coordinates

Click For Summary
The discussion centers on evaluating an integral in spherical coordinates, specifically regarding the alignment of vectors during integration. Participants clarify that while the x-vector changes, it can be fixed in relation to the z-axis due to spherical symmetry, allowing for easier calculations. The inclination angle's rotation around the z-axis is emphasized as a key factor in simplifying the integral. There is consensus that aligning the ξ direction with the z-axis makes the integral more manageable, although theoretically, any axis could be chosen. The conversation highlights the importance of understanding vector alignment in the context of spherical coordinates for effective integration.
aaaa202
Messages
1,144
Reaction score
2
I recently had to do an integral like the one in the thread below:
http://math.stackexchange.com/quest...-of-radial-function-without-bessel-and-neuman
The problem I had was also evaluating the product and I am quite sure that the answer in the thread is the one I need. I just don't understand it fully. They say we fix our x-vector such that its angle with the z-axis is the same as its dot product with the other vector. But isn't x an everchanging vector? I meant we are integrating over it. What am I missing?
 
Physics news on Phys.org
The integral will produce a function of vector ξ. For any given ξ, because of the spherical symmetry elsewhere, you can rotate your coordinates to align ξ with one of the axes. In this case, z was chosen.
 
Ahh I get it now. Though I don't see which other axis you could align it with? The inclination angle does not rotate around the x or y axis. The special reason we can use the z-axis is that the inclination azimuthal angle rotates around it. Do you not agree?
 
aaaa202 said:
Ahh I get it now. Though I don't see which other axis you could align it with? The inclination angle does not rotate around the x or y axis. The special reason we can use the z-axis is that the inclination azimuthal angle rotates around it. Do you not agree?

In principle you could choose the ξ direction for any of the axes, but maybe only choosing it as the z-axis makes the integral amenable.
 
Question: A clock's minute hand has length 4 and its hour hand has length 3. What is the distance between the tips at the moment when it is increasing most rapidly?(Putnam Exam Question) Answer: Making assumption that both the hands moves at constant angular velocities, the answer is ## \sqrt{7} .## But don't you think this assumption is somewhat doubtful and wrong?

Similar threads

  • · Replies 7 ·
Replies
7
Views
1K
  • · Replies 24 ·
Replies
24
Views
3K
  • · Replies 7 ·
Replies
7
Views
2K
Replies
3
Views
2K
  • · Replies 3 ·
Replies
3
Views
3K
  • · Replies 4 ·
Replies
4
Views
2K
  • · Replies 2 ·
Replies
2
Views
2K
  • · Replies 1 ·
Replies
1
Views
1K
  • · Replies 9 ·
Replies
9
Views
2K
  • · Replies 4 ·
Replies
4
Views
9K